Le site "intégralsbot"

24

Réponses

  • Gebrane: Ce qui est motivant c'est de faire les calculs soi-même et d'explorer le domaine (on y fait parfois de belles rencontres si je puis dire)
    Grimper un haut sommet par le chemin des touristes cela représente une balade qui n'est pas à la portée de tout le monde sans doute. Mais grimper le même sommet par la face nord c'est nettement plus grisant. Cela t'explique pourquoi parfois je ne me contente pas de suivre le chemin "touristique" en matière de calcul intégral et que j'aime rajouter des obstacles (pas d'intégrale doubles, pas de dérivation sous le signe intégral, par exemple). B-)-
  • Quentino37: quand à droite de l'intégrale il n'y a pas un signe égal avec une valeur à droite, généralement je ne me lance pas dans des calculs*. Je ne veux pas passer des heures à chercher pour qu'au bout de deux jours celui ou celle qui a proposé ce calcul se rende compte qu'il y avait une erreur d'énoncé, ou qu'il/elle avoue que l'intégrale a été créée au hasard ou presque et n'a donc pas de forme close remarquable.

    *: si l'AMM propose un tel calcul je suis en confiance qu'il y a une valeur remarquable à obtenir.
  • Gebrane: Vous restez quand même super trop ultra fort :)

    PS: Je ne crois pas que la langue française contient des adjectifs pouvant décrire à quel point vous êtes trop fort :)

    PS2: j'ai encore écrit un message en même temps que quelqu'un d'autre... :)
    Je suis donc je pense 
  • FDP: j'avoue que j'ai juste pris des fonctions au hasard et j'ai créé avec une intégrale :)
    Je suis donc je pense 
  • Quentino37:Quand j'étais adolescent j'étais fasciné par ces formules élégantes (que je ne comprenais pas vraiment à l'époque) comme celle-ci:

    \begin{align}\int_0^\infty \frac{\sin x}{x}dx=\frac{\pi}{2}\end{align}
    (je crois me rappeler que cette formule, ou une autre formule remarquable, était reproduite sur un de mes cahiers de brouillon de collège)
    Puis, en 2011 quelqu'un sur le forum a demandé un calcul d'intégral, pas trivial. J'ai eu une intuition*, j'ai tiré le bout de ficelle de la pelote et tout s'est bien agencé pour produire une solution. Depuis ce temps-là, je crois que j'ai approfondi ma passion pour ces calculs (totalement inutiles). Je crois que j'ai franchi une étape supplémentaire quand j'ai rejoint MathExchange en 2015. (sur ce board, je crois que je n'ai répondu à ce jour qu'à des questions tournant autour du calcul intégral voire quelques calculs de séries). Voilà comment on se perfectionne dans ce domaine (complètement inutile).

    *: je sais depuis qu'on pouvait faire plus "naturellement" ce calcul (c'est pénible cela dit).
    Il s'agissait de calculer $\displaystyle \int_0^1 \frac{\arctan\left(\frac{x(1-x)}{2-x}\right)}{x}dx$
    Intégrale proposée au calcul par un mathématicien américain sur une liste de diffusion au début des années 2000 (2007 je crois). La personne qui demandait n'avait pas de preuve.
    Je lui ai demandé par courriel d'où il tenait cette intégrale, il m'a dit l'avoir vue sur un site internet disparu. Je n'ai pas été capable de remonter la piste plus en avant. Je me disais que si cette intégrale avait une histoire il y avait peut-être
    de belles rencontres à faire dans son halo.

    PS:
    Ce type de calculs était un domaine complètement délaissé par les mathématiciens sérieux (c'est totalement inutile) depuis sans doute la fin du XIXème siècle. Puis j'ai l'impression qu'il y a à nouveau un engouement depuis le début des années 2000 (sans doute lié à l'essor de logiciels comme Maple,Mathematica,Maxima,...)
    Si on s'intéresse à ce domaine voici des livre qu'il faut avoir dans sa bibliothèque:

    1) Inside Interesting Integrals, de Paul J Nahin*
    2) Irresistible Integrals, de George Boros (RIP), Victor Moll.
    3) (Almost) Impossible Integrals, Sums, and Series de Cornel Ioan Valean.
    4) Limits, Series, and Fractional Part Integrals de Ovidiu Furdui.

    *: je viens de découvrir qu'il y a une nouvelle version (2020)
  • @Fdp je pensais que ta passion pour les intégrales était beaucoup plus ancienne.
  • Raoul.S: Elle n'était pas aussi intense, c'était épisodique.
    Je me souviens avoir passé 20 heures presque non stop (sans sommeil) sur un calcul d'intégral entre 2015 et aujourd'hui. :-D Chose que je ne faisais pas dans le passé.
    A la fin des années 90, j'étais passionné par certains aspects de l'informatique (j'ai passé beaucoup d'heures à coder avec tasm/tlink/fasm/masm/debug* ;) ) et j'étais accro' au site http://www.caesum.com/game/ (il m'a fallu plus de 10 ans pour compléter tous les problèmes)

    *: suite à un crash disque il ne me reste rien de tout ce travail. 8-)
  • FDP as tu réussi le calcul de $\int _0^1\frac{\left(\arctan x\right)^2}{x}dx$ avec ton cahier de charge
    Le 😄 Farceur


  • raoul, la personne qui m'a fait aimer le calcul intégral c'est Nancy
    Le 😄 Farceur


  • En effet, elle a des arguments très convaincants.
  • Gebrane: Non* Mais je pense que je peux tout de même calculer cette intégrale sans série de Fourier.

    *: je ne crois pas trop que ce soit possible.
  • Ok fixons les conditions, on élimine seulement l'analyse de Fourier ?
    Le 😄 Farceur


  • mouahaha gebrane, Nancy oui oui je la connais (que sur YouTube...), diplômée du MIT etc. mais je ne regardais pas ses vidéos pour les maths 8-)


    PS. Chaurien a l'air de l'apprécier également...
  • Pourquoi les profs de maths ne sont pas comme elle, elle explique bien les choses d'une manière claire et sublime
    Le 😄 Farceur


  • Gebrane a écrit:
    Pourquoi les profs de maths ne sont pas comme elle
    Un certain nombre de profs sont des hommes. B-)-
  • gebrane a écrit:
    Pourquoi les profs de maths ne sont pas comme elle...

    De nos jours tout est possible mais cela reste très onéreux, surtout si tu es barbu avec quelques kilos de trop B-)-
  • Une intégrale beaucoup plus facile à calculer, mais que j'aime bien:

    $\displaystyle \int_0^\infty \frac{\ln x}{1+x^3}dx$

    On admet que $\displaystyle \int_0^1 \frac{\ln x}{x-1}dx=\dfrac{\pi^2}{6}$. Ce qui permet de calculer l'intégrale ci-dessus sans utiliser d'intégrales doubles, sans dériver sous le signe intégral, sans développer en série.

    PS: J'ai effectué le calcul mais pour laisser chercher ceux qui voudraient le faire, j'ai repeint en blanc ce qui suit.
    (Edit: ceux qui avaient eu envie de se frotter à ce calcul l'ont déjà fait, les calculs sont désormais visibles)
    \begin{align} \int_0^\infty \frac{\ln x}{1+x^3}dx&= \int_0^1 \frac{\ln x}{1+x^3}dx+\underbrace{ \int_1^\infty \frac{\ln x}{1+x^3}dx}_{y=\frac{1}{x}}\\
    &= \int_0^1\frac{(1-x)\ln x}{1+x^3}dx\\
    &\overset{\text{IPP}}=\left[\left(\frac{2}{3}\ln(1+x)-\frac{1}{3}\ln(1-x+x^2)\right)\ln x\right]_0^1-\int_0^1 \frac{\frac{2}{3}\ln(1+x)-\frac{1}{3}\ln(1-x+x^2)}{x}dx\\
    &=-\frac{2}{3}\int_0^1\frac{\ln(1+x)}{x}dx+\frac{1}{3}\underbrace{\int_0^1\frac{\ln(1+x^3)}{x}dx}_{y=x^3}-\frac{1}{3}\int_0^1\frac{\ln(1+x)}{x}dx\\
    &=-\frac{8}{9}\int_0^1\frac{\ln(1+x)}{x}dx\\
    &\overset{\text{IPP}}=-\frac{8}{9}\Big[\ln x\ln(1+x)\Big]_0^1+\frac{8}{9}\int_0^1\frac{\ln x}{1+x}dx\\
    &=\frac{8}{9}\int_0^1\frac{\ln x}{1+x}dx\\
    &=\frac{8}{9}\int_0^1\frac{\ln x}{1-x}dx-\frac{8}{9}\underbrace{\int_0^1\frac{2x\ln x}{1-x^2}dx}_{y=x^2}\\
    &=\frac{8}{9}\int_0^1\frac{\ln x}{1-x}dx-\frac{8}{18}\int_0^1\frac{\ln x}{1-x}dx\\
    &=\frac{4}{9}\int_0^1\frac{\ln x}{1-x}dx\\
    &==\frac{4}{9}\times -\zeta(2)\\
    &=\frac{4}{9}\times -\frac{\pi^2}{6}\\
    &=\boxed{-\dfrac{2\pi^2}{27}}
    \end{align}
  • Pour le calcul de $\displaystyle \int_0^1 \frac{\arctan x\ln x}{1+x^2}dx$ voir le calcul de $I-J$ ici.
    Peut-on faire plus simple sans utiliser de séries de Fourier et de fonctions de Clausen (un moyen de planquer les séries de Fourier sous le tapis)?

    PS: Il y a peut-être un peu plus simple (mais il faut que je vérifie mes calculs)
  • FDP, je ne comprends pas ta phrase '' Peut-on faire plus simple sans utiliser de séries de Fourier et de fonctions de Clausen (un moyen de planquer les séries de Fourier sous le tapis)? ''

    Est ce que tu vois ce-dessous des séries de Fourier ?

    On pose

    $\displaystyle K=\int_0^{1}\frac{\ln x\arctan x}{1+x^2} \,dx$

    et pour $x\in [0;1]$, je pose à la FDP

    $\begin{align}R(x)&=\int_0^x\frac{\ln t}{1+t^2}\,dt\\
    &=\int_0^1 \frac{x\ln(xt)}{1+t^2x^2}\,dt
    \end{align}$

    On a , $\displaystyle R(0)=0,R(1)=-\text{G}$.

    Par ipp

    $\begin{align}K&=\Big[R(x)\arctan x\Big]_0^1-\int_0^1 \frac{R(x)}{1+x^2}\,dx\\
    &=-\frac{\pi}{4}\text{G}-\int_0^1 \int_0^1 \frac{x\ln(xt)}{(1+t^2x^2)(1+x^2)}\,dt\,dx \\
    &=-\frac{\pi}{4}\text{G}-\int_0^1 \int_0^1 \frac{x\ln t}{(1+t^2x^2)(1+x^2)}\,dt\,dx-\int_0^1 \int_0^1 \frac{x\ln x}{(1+t^2x^2)(1+x^2)}\,dt\,dx\\
    &=-\frac{\pi}{4}\text{G}-\frac{1}{2}\int_0^1 \left[\frac{\ln t}{1-t^2}\times \ln\left(\frac{1+x^2}{1+t^2x^2}\right)\right]_{x=0}^{x=1}\,dt-\int_0^1 \Big[\frac{\arctan(tx)\ln x}{1+x^2}\Big]_{t=0}^{t=1} \,dx\\
    &=-\frac{\pi}{4}\text{G}-\frac{1}{2}\int_0^1 \frac{\ln\left( \frac{2}{1+t^2}\right)\ln t}{1-t^2}\,dt-K
    \end{align}$

    donc

    $\begin{align} K&=-\frac{\pi}{8}\text{G}-\frac{1}{4}\int_0^1 \frac{\ln\left( \frac{2}{1+t^2}\right)\ln t}{1-t^2}\,dt\\
    &=-\frac{\pi}{8}\text{G}-\frac{\ln 2}{4}\int_0^1 \frac{\ln t}{1-t^2}\,dt+\frac{1}{4}\int_0^1 \frac{\ln(1+t^2)\ln t}{1-t^2}\,dt
    \end{align}$

    On note

    $\displaystyle L=\int_0^1 \frac{\ln(1+t^2)\ln t}{1-t^2}\,dt$

    pour $x\in [0;1]$ on définit

    $\begin{align}S(x)&=\int_0^x\frac{\ln t}{1-t^2}\,dt\\
    &=\int_0^1 \frac{x\ln(tx)}{1-t^2x^2}\,dt
    \end{align}$

    Par ipp

    $\begin{align}L&=\Big[S(x)\ln(1+x^2)\Big]_0^1 -\int_0^1 \int_0^1\frac{2x^2\ln(tx)}{(1+x^2)(1-t^2x^2)}\,dt\,dx\\
    &=S(1)\ln 2-\int_0^1 \int_0^1 \frac{2x^2\ln t}{(1+x^2)(1-t^2x^2)}\,dt\,dx-\int_0^1 \int_0^1\frac{2x^2\ln x}{(1+x^2)(1-t^2x^2)}\,dt\,dx\\
    &=S(1)\ln 2-\int_0^1 \left[-\frac{t\ln t}{1+t^2}\ln\left(\frac{1+tx}{1-tx}\right)+\frac{\ln t}{t}\ln\left(\frac{1+tx}{1-tx}\right)-\frac{2\arctan x \ln t}{1+t^2}\right]_{x=0}^{x=1}\,dt-\\
    &\int_0^1 \left[\frac{x\ln x}{1+x^2}\ln\left(\frac{1+tx}{1-tx}\right)\right]_{t=0}^{t=1}\,dx\\
    &=S(1)\ln 2-\int_0^1 \left[\frac{\ln t}{t}\ln\left(\frac{1+tx}{1-tx}\right)-\frac{2\arctan x \ln t}{1+t^2}\right]_{x=0}^{x=1}\,dt\\
    &=S(1)\ln 2-\int_0^1 \frac{\ln t}{t}\ln\left(\frac{1+t}{1-t}\right)\,dt+\frac{\pi}{2}\int_0^1 \frac{\ln t}{1+t^2}\,dt\\
    &=S(1)\ln 2-\int_0^1 \frac{\ln t}{t}\ln\left(\frac{1+t}{1-t}\right)\,dt-\frac{1}{2}\pi\text{G}
    \end{align}$

    Soit ,

    $\displaystyle M=\int_0^1 \frac{\ln t}{t}\ln\left(\frac{1+t}{1-t}\right)\,dt$

    Par ipp

    $\begin{align}M&=\Big[\frac{1}{2}\ln^2 t \ln\left(\frac{1+t}{1-t}\right)\Big]_0^1-\int_0^1 \frac{\ln^2 t}{1-t^2}\,dt\\
    &=-\int_0^1 \frac{\ln^2 t}{1-t^2}\,dt\\
    \end{align}$

    donc

    $\displaystyle M=-\frac{7}{4}\zeta(3)$

    donc

    $\displaystyle L=S(1)\ln 2+\frac{7}{4}\zeta(3)-\frac{1}{2}\pi\text{G}$

    donc

    $\begin{align}K&=-\frac{\pi}{8}\text{G}-\frac{\ln 2}{4}S(1) +\frac{1}{4}L\\
    &=-\frac{\pi}{8}\text{G}-\frac{\ln 2}{4}S(1) +\frac{1}{4}\left(S(1)\ln 2+\frac{7}{4}\zeta(3)-\frac{1}{2}\pi\text{G}\right)\\
    &=\frac{7}{16}\zeta(3)-\frac{1}{4}\pi\text{G}
    \end{align}$
    Le 😄 Farceur


  • FDP tu n'as pas répondu
    Le 😄 Farceur


  • Gebrane:

    Il me semble que tu as recopié ce que j'ai mis en lien ci-dessus*. Tu me poses une question qui a une réponse évidente (elle l'est au moins évidente pour moi). NON.

    Et, je n'ai pas trouvé plus simple (avec le cahier des charges indiqué) à ce jour.
    Autrement, si on ne s'interdit pas les séries de Fourier on fait le changement de variable $x=\tan t$.

    *: je t'en remercie, j'avais eu la paresse de le faire.
  • FDP j'ai reproduis ton travail pour m indiquer l'endroit où tu as utilisé les séries de Fouruer. Car tu t'es demandé comment faire, avec ton cahier de charge. Mais cette preuve respecte le cahier de charge, donc j'ai trouvé ta demande bizarre.
    Le 😄 Farceur


  • Bravo FdP pour cette $\displaystyle \int_0^{+ \infty} \frac{\ln x}{1+x^3}dx$.
    Moi j'étais arrivé à $\displaystyle \int_0^1\frac{(1-x)\ln x}{1+x^3}dx$, et j'avais fait la décomposition en élément simples, mais sans pouvoir aller plus loin : impasse. La bonne idée que je n'ai pas eue c'est l’intégration par parties.
    Mais une fois arrivé à $\displaystyle -\frac 89 \int_0^1\frac{\ln(1+x)}{x}dx$, pourquoi s'obliger à passer à $\displaystyle \int_0^1\frac{\ln x}{x-1}dx$ ? Celle-ci n'est pas une sorte de mètre-étalon, que je sache.
    .Mieux aurait valu donner comme suggestion : $\displaystyle \int_0^1\frac{\ln(1+x)}{x}dx=\frac {\pi^2}{12}$, ce qui se trouve tout de suite avec un développement en série entière et un argument d'interversion immédiats. Ce n'est d'ailleurs pas une intégrale impropre. Et c'est terminé.
    C'est vraiment une belle intégrale. Encore bravo, et bonne soirée.
    Fr. Ch.
  • Gebrane: je crois qu'il y a une quiproquo.

    J'ai écrit plus haut:
    Peut-on faire plus simple sans utiliser de séries de Fourier et de fonctions de Clausen (un moyen de planquer les séries de Fourier sous le tapis)?

    Je posais une question à laquelle je n'ai pas de réponse. La question n'était pas: y-a-t-il une solution sans séries de Fourier (ou fonctions de Clausen)? Parce que j'avais déjà une réponse à cette question.

    Mais la solution développée (qui satisfait à peu près* à mon cahier des charges) est trop longue à mon goût. La première intégration par parties (qui est un peu sportive) ne suffit pas pour finir le calcul, il en faut une deuxième et cela m'ennuie (mais je ne vois pas comment faire autrement à ce jour en utilisant une méthode similaire).

    *: il y a une utilisation d'intégrales doubles. J'aurais bien aimé la proscrire.
  • Ok j avais mal compris ta demande
    Le 😄 Farceur


  • Chaurien:

    Pour moi quelques intégrales sont bien des intégrales étalons. J'avais fixé des règles assez restrictives pour faire ce calcul (on peut sûrement l'effectuer par un calcul d'intégrales par la méthode des résidus) donc je me devais de les respecter. D'autant plus, qu'à ce jour, personne à ma connaissance n'a jamais réussi à montrer que $\displaystyle \int_0^1 \frac{\ln x}{x-1}dx=\frac{\pi^2}{6}$ sans l'utilisation: 1) d'intégrales multiples 2) séries (séries de Fourier....) 3) Dérivation d'une intégrale à paramètre.

    Par ailleurs, la vraie question pour moi, n'est pas de montrer qu'on peut exprimer $\displaystyle \int_0^\infty \frac{\ln x}{1+x^3}dx$* en fonction du produit d'un nombre rationnel fois $\pi^2$ $(1)$ mais de montrer que cette intégrale est égale à un rationnel fois $\displaystyle \int_0^1 \frac{\ln x}{x-1}dx$. Mais pour que la question soit plus "sexy" il faut la formuler sous la forme $(1)$. B-)-


    Dans le livre Inside Interesting Integrals, de Paul J Nahin (deuxième édition), cet auteur explique que G.H Hardy aurait calculé cette intégrale en développant en série la fonction $\dfrac{1-x}{1+x^3}$. (j'ai bien trouvé dans quelle revue a été publiée ce problème en $1899$ mais il ne me semble pas avoir vu d'utilisation de série).
    Ce qui fait qu'on va obtenir que cette intégrale est égale à un rationnel fois $\zeta(2)$ (en prenant comme définition de $\zeta(2)$ sa définition habituelle sous forme de série) et, à nouveau, pour faire plus "sexy" il faudra bien qu'on utilise que $\zeta(2)=\frac{\pi^2}{6}$. égalité qui n'est pas triviale.

    *: cette intégrale est sûrement calculable en utilisant la fonction bêta d'Euler et une dérivation sous le signe intégrale d'une intégrale à paramètre.
  • Voilà comment je calculerais $\int_0^1 \frac{\ln(x)}{x-1}dx$
    On à $\displaystyle
    \ln(1+x)=x-\frac{x^{2}}{2}+\frac{x^{3}}{3}-\cdots+(-1)^{n+1}\frac{x^{n}}{n}+\ldots $
    Donc $\displaystyle
    \ln(x)=(x-1)-\frac{(x-1)^{2}}{2}+\frac{(x-1)^{3}}{3}-\cdots+(-1)^{n+1}\frac{(x-1)^{n}}{n}+\ldots$
    et $\displaystyle
    \frac{\ln(x)}{x-1}=1-\frac{(x-1)^{1}}{2}+\frac{(x-1)^{2}}{3}-\cdots +(-1)^{n+1}\frac{(x-1)^{n-1}}{n}+\ldots$
    et $\displaystyle
    \int \frac{\ln(x)}{x-1}dx=x-\frac{(x-1)^{2}}{2×2}+\frac{(x-1)^{3}}{3×3}-\cdots+(-1)^{n+1}\frac{(x-1)^{n}}{n^{2}}+\ldots$
    Donc
    \begin{align*}
    \int_0^1 \frac{\ln(x)}{x-1}dx&=1 - \Big(-\frac{(-1)^{2}}{2×2}+\frac{(-1)^{3}}{3×3}-\cdots+(-1)^{n+1}\frac{(-1)^{n}}{n^{2}}+\ldots\Big) \\
    &=1-\Big(-\sum_{n=2}^{\infty} \frac{1}{n^{2}}\Big) \\
    &=\sum_{n=1}^{\infty} \frac{1}{n^{2}}=\frac{\pi^{2}}{6}.

    \end{align*} Finallement
    \[\int_0^1 \frac{\ln(x)}{x-1}=\frac{\pi^{2}}{6}.

    \] Et voila c'est prouvé !!!

    PS : je suis sur d'avoir fait une faute de frappe quelque part...
    Je suis donc je pense 
  • Par curiosité, j'ai demandé à l'esclave électronique ce qu'on trouve pour $\displaystyle \int_{0}^{+\infty }\frac{\ln t}{1+t^{a}}dt$, $a>1$.
    Il semblerait que ce soit : $\displaystyle -(\frac{\pi }{a})^{2}\frac{\cos \frac{\pi }{a}}{\sin ^{2}\frac{\pi }{a}}$.
    Mais comment le prouver ?
    Bonne nuit.
    Fr. Ch.
  • Chaurien:

    Il faut très certainement considérer la fonction $\displaystyle F(s)=\int_0^\infty \frac{t^s}{1+t^a}dt$.
    (exprimable comme fonction bêta d'Euler, $s$ a vocation à être proche de $0$)

    En fait, c'est curieux que tu demandes ça car je crois que le problème posé dans la revue de $1899$ dont je parlais était plus général que le cas $a=3$ et utilisait je pense une expression de la fonction $F$ (de mémoire).
  • Bonjour,

    @Chaurien :

    On part de $\displaystyle \int_I {\ln t \over 1+t^a} dt$ pour $\displaystyle a>1$ et $I=[0,+\infty[.$

    D'abord on vire cet affreux dénominateur : $\displaystyle t^a = y$ donne $\displaystyle {1\over a^2} \int_I {y^{1/a-1}\ln y \over 1+y} dy$,
    puis on vire cet affreux logarithme : $\displaystyle {1\over a^2} \int_I {y^{1/a-1} y^b \over 1+y} dy$ pour $b \geq 0.$

    On reconnaît une fonction bêta $\star$ $\displaystyle {1\over a^2} B({1\over a}+b, 1-{1\over a}-b) =\displaystyle {\pi \over \sin(\pi ({1\over a}+b))}.$

    La dérivée partielle par rapport à $b$ prise en $0$ donne le résultat.

    $\star$ Comme il est tard, je l'écris $\displaystyle B(x,y) = \int_I {t^{x-1} \over (1+t)^{x+y}} dt.$
  • Comment fait-on pour virer cet affreux logarithme ? :)o :)
    Je suis donc je pense 
  • Bonjour,

    Pour virer un logarithme au numérateur on le remplace par une puissance (ici en $b$), puis on dérive sous le signe somme par rapport à $b$ et on prend en $0$ puisque ${\partial \over \partial b}x^b\mid_0 = \ln x.$
  • Une copie de "la solution" proposée par G.H Hardy.

    On la trouve dans ce journal, The educational times, 1899, march, p160

    Je ne comprends pas très bien la mention "The rest in volume".122596
  • Bonsoir,

    > Je ne comprends pas très bien la mention "The rest in volume".

    La suite dans le numéro suivant, qui fait partie du volume ... probablement.

    Cordialement,

    Rescassol
  • Rescassol: J'ai parcouru toute la partie problèmes mathématiques de ce volume et je n'ai rien vu d'autre concernant ce problème. :-D
    (les problèmes sont surtout des problèmes de géométrie, même si on trouve des problèmes divers et variés).
  • Vous êtes chauds pour montrer que $\displaystyle \int_0^\infty \frac{\ln^2 x}{1+x^3}dx=\dfrac{10\pi^3}{81\sqrt{3}}$ B-)-
  • Oui je suis chaude pour le montrer :-D
    Sauf erreur ton intégrale s’écrit $\int_0^1 \frac{\log^2 x}{x^2-x+1}\mathrm dx$, il fait tard

    PS J'ai constaté dernièrement que tu doutes sur mon genre H ou F
    Le 😄 Farceur


  • Gebrane:

    Ok, pour le genre.

    Je ne suis pas sûr que cela soit une bonne idée de faire cette simplification. En l'état, on peut utiliser la même technique de calcul que celle décrite par YvesM plus haut (mais il faut calculer une dérivée seconde).


    Quentino37:
    $\displaystyle \ln(1-x)=-x-\frac{x^2}{2}-\frac{x^3}{3}-\frac{x^4}{4}-...$
    Ce développement en série entière ne converge (pour des valeurs réelles) que sur l'intervalle $[-1,1[$
    Si tu fais un changement de variable dans la formule tu changes l'intervalle où la série converge.
    Si on veut intégrer sur $[0;1]$ en développant en série entière il faut développer la fonction considérée sur ce même intervalle.
    Par ailleurs, la présence d'un facteur $\ln^n(x)$ n'est pas un problème car on connait la valeur des intégrales $\displaystyle \int_0^1 x^m\ln^n xdx$ ($m,n$ des entiers positifs) qu'on devra calculer si on développe en série entière sur $[0;1]$ la fonction $f$ de l'intégrande dans l'intégrale $ \int_0^1 f(x)\ln^n xdx$
  • \[\ln(1+x)=x-\frac{x^{2}}{2}+\frac{x^{3}}{3}-\cdots + (-1)^{n+1}\frac{x^{n}}{n}+\ldots
    \] ça ne converge que sur l'intervalle $]-1; 1]$ (sauf erreur) du coup
    \[\ln(x)=(x-1)-\frac{(x-1)^{2}}{2}+\frac{(x-1)^{3}}{3}-\cdots + (-1)^{n+1}\frac{(x-1)^{n}}{n}+\ldots
    \] et ça converge sur l'intervalle $]0; 2]$, or on veux intégrer sur l'intervalle $[0; 1]$ (sauf erreur).
    Ou est l'erreur Fin De Partie ?
    Je suis donc je pense 
  • Quentino37:
    Je pense qu'il n'y en a pas (dans ton message précédent tu ne donnais aucun intervalle de convergence pour les séries écrites).
    Mais que fais-tu de cette série ici? B-)-
  • J'aurais préciser les intervalles de convergence ? :-)
    Je suis donc je pense 
  • Je reviens sur cette intégrale $\displaystyle \int_{0}^{1}\frac{\ln t}{t-1}dt$ qui semble intéresser particulièrement FdP. Je trouve bien superflu d'aller chercher séries de Fourier, integrales doubles, dérivation sous le signe d'intégrale.
    Comme dit FdP, on a, pour $n\in \mathbb{N}$ et $p\in \mathbb{N}$ : $J_{n,p}=\int_{0}^{1}t^{n}(\ln t)^{p}dt=(-1)^{p}\frac{p!}{(n+1)^{p+1}}$, par une récurrence très simple sur $p$, basée sur une IPP.
    Alors : $\displaystyle \int_{0}^{1}\frac{\ln t}{t-1}dt=\int_{0}^{1}\overset{+\infty }{\underset{n=0}{\sum }}(-t^{n}\ln t)dt=-\overset{+\infty }{\underset{n=0}{\sum }}%
    \int_{0}^{1}t^{n}\ln tdt=\overset{+\infty }{\underset{n=0}{\sum }}\frac{1}{%
    (n+1)^{2}}=\zeta (2)$.
    MAIS... en toute rigueur, il faudrait d'abord s'assurer de la convergence de l'intégrale avant de la calculer, et même si l'on trouve ce souci exagéré, il faut justifier l'interversion série-intégrale, exigence dont s'affranchissent souvent les livres anglo-saxons... et certains de leurs lecteurs ;-).
    Malheureusement, la convergence de la série $\displaystyle \overset{+\infty }{\underset{n=0}{\sum }} (-t^{n}\ln t)$ n'est pas normale sur $[0,1]$.
    Heureusement, nous avons affaire à une série géométrique, dont le reste a une expression explicite très simple, de par la formule bien connue : $\displaystyle \frac{t^{n+1}-1}{t-1}=\overset{n}{\underset{k=0}{\sum }}t^{k}$. Il en résulte :
    $\displaystyle \int_{0}^{1}\frac{\ln t}{t-1}dt=\int_{0}^{1}\overset{n}{\underset{k=0}{\sum
    }}(-t^{k}\ln t)dt+\int_{0}^{1}\frac{t^{n+1}\ln t}{t-1}dt=\overset{n}{%
    \underset{k=0}{\sum }}\frac{1}{(k+1)^{2}}+R_{n}$, avec : $\displaystyle R_{n}=\int_{0}^{1}t^{n}\frac{t\ln t}{t-1}dt$.
    Pour une somme finie, l'interversion va de soi.
    La fonction : $t\mapsto \frac{t\ln t}{t-1}$ est continue et positive sur $[0,1]$. Il existe donc une constante $M>0$ telle que, pour tout $t \in [0,1]$ : $0\leq \frac{t\ln t}{t-1}\leq M$, et par suite : $0\leq R_{n}\leq \int_{0}^{1}Mt^{n}dt=\frac{M}{n+1}$, d'où $R_n \rightarrow 0$, ce qui règle la question.
    Cette démonstration est utilisable dans les classes où les suites et séries de fonctions ne figurent pas au programme.
    On peut saluer l'apparition du nombre $\pi$ dans un calcul d'intégrale portant sur un logarithme, qui illustre une certaine unité des mathématiques.
    Bonne journée.
    Fr. Ch.
  • A propos du logarithme.

    Puisqu'on a pour $x\in]0,1[$, $\displaystyle \ln(1-x)=-x-\dfrac{x^2}{2}-\dfrac{x^3}{3}-...$ et $\displaystyle \ln(1+x)=x-\dfrac{x^2}{2}+\dfrac{x^3}{3}-...$
    On a donc $\displaystyle -\dfrac{1}{2}\ln\left(\dfrac{1-x}{1+x}\right)=x+\dfrac{x^3}{3}+\dfrac{x^5}{5}+\dfrac{x^7}{7}+...$

    On fait le changement de variable $y=\dfrac{1-x}{1+x}$ et on a donc pour tout $y\in ]0;1]$,
    $\displaystyle -\dfrac{1}{2}\ln y=\left(\dfrac{1-y}{1+y}\right)+\frac{1}{3}\left(\dfrac{1-y}{1+y}\right)^3+\frac{1}{5}\left(\dfrac{1-y}{1+y}\right)^5+...$
  • Chaurien:

    Le lien entre $\displaystyle \int_0^1 \frac{\ln x}{x-1}dx$ et $\zeta(2)$ est élémentaire j'en suis bien d'accord mais c'est le lien entre $\zeta(2)$ et $\pi^2$ qui ne l'est pas*. On a tendance à passer de l'une à l'autre de ces trois expressions: $\zeta(2),\displaystyle \int_0^1 \frac{\ln x}{x-1}dx,\dfrac{\pi^2}{6}$ en oubliant la difficulté de passer de l'une à l'autre. 8-)

    *: autrement Euler aurait peut-être consacré sa vie à en faire autre chose que faire des mathématiques.
  • Additif à la fin de mon précédent message. Au lieu d'invoquer la continuité de la fonction $t\mapsto \frac{t\ln t}{t-1}$ sur $[0,1]$, on peut observer que pour tout $t>0$
    on a : $\ln t \le t-1$, d'où : $\ln \frac 1t \le \frac 1t -1$, et par suite pour $ t \in ]0,1[ $ : $\frac { t \ln t}{t-1} \le 1$.
  • FDP. En utilisant le développement en série de Taylor de $\frac{\sin(x)}{x}$ et en utilisant le fait que c'est un polynôme on peut le transformer en produit infini et en développant le produit infini on obtient que le coefficient de $x^2$ est \[\sum_{n=1}^{\infty}\frac{1}{\pi^{2}n^{2}}\qquad\text{d'où}\qquad
    \frac{1}{3!}=\sum_{n=1}^{\infty}\frac{1}{\pi^{2}n^{2}},
    \] et on en déduit que $\zeta(2)=\frac{\pi^{2}}{6}$
    (c'est une preuve plutôt simple ! par contre je ne sais pas comment Euler à eu cette idée géniale...).

    PS : c'est en essayant de comprendre pourquoi $\zeta(2)=\frac{\pi^{2}}{6}$ que j'ai beaucoup appris, et merci à Mickaël Launay pour cette video qui m'a fait me poser beaucoup de questions... :-)
    Je suis donc je pense 
  • FdP, la formule : $ \displaystyle \overset{+\infty }{\underset{n=1}{\sum }}\frac{1}{n^{2}}=\frac {\pi^2}6 $, on en a parlé plusieurs fois sur ce forum, et tu as dans tes archives un grand nombre de démonstrations, dont certaines sont élémentaires et peuvent être traitées à bac+1, ce que j'ai fait plusieurs fois avec mes élèves . En particulier celle des frères Yaglom, avec les polynômes, dont tu as trouvé la première publication, qui remonte aux années 1950. Il est inutile de soulever des difficultés à ce sujet.
    Bonne journée.
    Fr. Ch.
  • Petite curiosité.
    Si $\displaystyle A=\int_0^\infty \frac{\ln x}{1+x^3}dx,B=\int_0^\infty \frac{\ln^2 x}{x^3-1}dx,C=\int_0^\infty \frac{1}{1+x^3}dx$
    On peut montrer que $B=2AC=\dfrac{4\pi}{3\sqrt{3}}A$

    PS: On a que $\displaystyle \int_0^\infty \frac{1}{1+x^3}dx=\int_0^1 \frac{1}{1+x^3}dx+\underbrace{\int_1^\infty \frac{1}{1+x^3}dx}_{y=\frac{1}{x}}=\int_0^1 \frac{1}{1-x+x^2}dx=\left[\frac{2}{\sqrt{3}}\arctan\left(\frac{2x-1}{\sqrt{3}}\right)\right]_0^1=\dfrac{2\pi}{3\sqrt{3}}$
  • Quentino27, Euler a eu cette idée géniale, probablement parce que... il était un génie.
  • Chaurien: "élémentaire" signifie pour moi qui s'obtient par intégration(s) par parties et changement(s) de variables dans des fonctions usuelles. Dans ce sens-là, le lien entre $\displaystyle \int_0^1 \dfrac{\ln x}{x-1}dx$ et $\zeta(2)$ n'est pas réellement élémentaire (même si, comme tu l'as montré, cela ne nécessite pas de connaître beaucoup en analyse pour établir ce lien).
Connectez-vous ou Inscrivez-vous pour répondre.